A list of proofs of “Coherent topoi have enough points”Set-theoretic forcing over sites?What do coherent topoi have to do with completeness?Is it possible for a theorem to be constructive only in a non-constructive metatheory?The real numbers object in Sh(Top)Can the algebraic geometry of schemes be developed internally in topoi?Model existence theorem in topos theoryWhat is the geometric significance of fibered category theory in topos theory?

A list of proofs of “Coherent topoi have enough points”


Set-theoretic forcing over sites?What do coherent topoi have to do with completeness?Is it possible for a theorem to be constructive only in a non-constructive metatheory?The real numbers object in Sh(Top)Can the algebraic geometry of schemes be developed internally in topoi?Model existence theorem in topos theoryWhat is the geometric significance of fibered category theory in topos theory?













5












$begingroup$


For my research I would like to read all the known proofs of the very classical result "Coherent topoi have enough points", by Deligne.




Ref 1: D3.3.13 in Sketches of an Elephant




provides a very logic-rooted proof of the statement, I would like to see a more geometric or a more category theoretic proof.




Suggested by Christian Espindola




Ref 2: 7.44 in Topos Theory by Johnstone.



Ref 3: 9.11.3 in Sheaves in Geometry and Logic.











share|cite|improve this question











$endgroup$









  • 3




    $begingroup$
    The original proof by Deligne appeared in SGA4, but it might be a little bit hard to found: it is in the appendix of exposé VI, (which is part II)
    $endgroup$
    – Simon Henry
    8 hours ago















5












$begingroup$


For my research I would like to read all the known proofs of the very classical result "Coherent topoi have enough points", by Deligne.




Ref 1: D3.3.13 in Sketches of an Elephant




provides a very logic-rooted proof of the statement, I would like to see a more geometric or a more category theoretic proof.




Suggested by Christian Espindola




Ref 2: 7.44 in Topos Theory by Johnstone.



Ref 3: 9.11.3 in Sheaves in Geometry and Logic.











share|cite|improve this question











$endgroup$









  • 3




    $begingroup$
    The original proof by Deligne appeared in SGA4, but it might be a little bit hard to found: it is in the appendix of exposé VI, (which is part II)
    $endgroup$
    – Simon Henry
    8 hours ago













5












5








5


2



$begingroup$


For my research I would like to read all the known proofs of the very classical result "Coherent topoi have enough points", by Deligne.




Ref 1: D3.3.13 in Sketches of an Elephant




provides a very logic-rooted proof of the statement, I would like to see a more geometric or a more category theoretic proof.




Suggested by Christian Espindola




Ref 2: 7.44 in Topos Theory by Johnstone.



Ref 3: 9.11.3 in Sheaves in Geometry and Logic.











share|cite|improve this question











$endgroup$




For my research I would like to read all the known proofs of the very classical result "Coherent topoi have enough points", by Deligne.




Ref 1: D3.3.13 in Sketches of an Elephant




provides a very logic-rooted proof of the statement, I would like to see a more geometric or a more category theoretic proof.




Suggested by Christian Espindola




Ref 2: 7.44 in Topos Theory by Johnstone.



Ref 3: 9.11.3 in Sheaves in Geometry and Logic.








reference-request ct.category-theory topos-theory






share|cite|improve this question















share|cite|improve this question













share|cite|improve this question




share|cite|improve this question








edited 8 hours ago







Ivan Di Liberti

















asked 8 hours ago









Ivan Di LibertiIvan Di Liberti

2,0091 gold badge7 silver badges23 bronze badges




2,0091 gold badge7 silver badges23 bronze badges










  • 3




    $begingroup$
    The original proof by Deligne appeared in SGA4, but it might be a little bit hard to found: it is in the appendix of exposé VI, (which is part II)
    $endgroup$
    – Simon Henry
    8 hours ago












  • 3




    $begingroup$
    The original proof by Deligne appeared in SGA4, but it might be a little bit hard to found: it is in the appendix of exposé VI, (which is part II)
    $endgroup$
    – Simon Henry
    8 hours ago







3




3




$begingroup$
The original proof by Deligne appeared in SGA4, but it might be a little bit hard to found: it is in the appendix of exposé VI, (which is part II)
$endgroup$
– Simon Henry
8 hours ago




$begingroup$
The original proof by Deligne appeared in SGA4, but it might be a little bit hard to found: it is in the appendix of exposé VI, (which is part II)
$endgroup$
– Simon Henry
8 hours ago










1 Answer
1






active

oldest

votes


















5











$begingroup$

The proof you cite is certainly based in the completeness theorem for coherent logic, but that book contains a fully categorical proof of such a theorem, based on ideas of Joyal, so it can qualify as category-theoretic. It makes use of previous categorical constructions from other parts of the book.



For a more geometric proof you can refer to Deligne's original proof, which is actually exposed in detail in Johnstone's other book "Topos theory", section 7.4. It is actually essentially the same argument, though you can distinguish the geometric flavour here.



And there is of course the proof given in Maclane-Moerdijk "Sheaves in geometry and logic" which is based on Barr's theorem and is also quite geometric.






share|cite|improve this answer











$endgroup$

















    Your Answer








    StackExchange.ready(function()
    var channelOptions =
    tags: "".split(" "),
    id: "504"
    ;
    initTagRenderer("".split(" "), "".split(" "), channelOptions);

    StackExchange.using("externalEditor", function()
    // Have to fire editor after snippets, if snippets enabled
    if (StackExchange.settings.snippets.snippetsEnabled)
    StackExchange.using("snippets", function()
    createEditor();
    );

    else
    createEditor();

    );

    function createEditor()
    StackExchange.prepareEditor(
    heartbeatType: 'answer',
    autoActivateHeartbeat: false,
    convertImagesToLinks: true,
    noModals: true,
    showLowRepImageUploadWarning: true,
    reputationToPostImages: 10,
    bindNavPrevention: true,
    postfix: "",
    imageUploader:
    brandingHtml: "Powered by u003ca class="icon-imgur-white" href="https://imgur.com/"u003eu003c/au003e",
    contentPolicyHtml: "User contributions licensed under u003ca href="https://creativecommons.org/licenses/by-sa/3.0/"u003ecc by-sa 3.0 with attribution requiredu003c/au003e u003ca href="https://stackoverflow.com/legal/content-policy"u003e(content policy)u003c/au003e",
    allowUrls: true
    ,
    noCode: true, onDemand: true,
    discardSelector: ".discard-answer"
    ,immediatelyShowMarkdownHelp:true
    );



    );













    draft saved

    draft discarded


















    StackExchange.ready(
    function ()
    StackExchange.openid.initPostLogin('.new-post-login', 'https%3a%2f%2fmathoverflow.net%2fquestions%2f338862%2fa-list-of-proofs-of-coherent-topoi-have-enough-points%23new-answer', 'question_page');

    );

    Post as a guest















    Required, but never shown

























    1 Answer
    1






    active

    oldest

    votes








    1 Answer
    1






    active

    oldest

    votes









    active

    oldest

    votes






    active

    oldest

    votes









    5











    $begingroup$

    The proof you cite is certainly based in the completeness theorem for coherent logic, but that book contains a fully categorical proof of such a theorem, based on ideas of Joyal, so it can qualify as category-theoretic. It makes use of previous categorical constructions from other parts of the book.



    For a more geometric proof you can refer to Deligne's original proof, which is actually exposed in detail in Johnstone's other book "Topos theory", section 7.4. It is actually essentially the same argument, though you can distinguish the geometric flavour here.



    And there is of course the proof given in Maclane-Moerdijk "Sheaves in geometry and logic" which is based on Barr's theorem and is also quite geometric.






    share|cite|improve this answer











    $endgroup$



















      5











      $begingroup$

      The proof you cite is certainly based in the completeness theorem for coherent logic, but that book contains a fully categorical proof of such a theorem, based on ideas of Joyal, so it can qualify as category-theoretic. It makes use of previous categorical constructions from other parts of the book.



      For a more geometric proof you can refer to Deligne's original proof, which is actually exposed in detail in Johnstone's other book "Topos theory", section 7.4. It is actually essentially the same argument, though you can distinguish the geometric flavour here.



      And there is of course the proof given in Maclane-Moerdijk "Sheaves in geometry and logic" which is based on Barr's theorem and is also quite geometric.






      share|cite|improve this answer











      $endgroup$

















        5












        5








        5





        $begingroup$

        The proof you cite is certainly based in the completeness theorem for coherent logic, but that book contains a fully categorical proof of such a theorem, based on ideas of Joyal, so it can qualify as category-theoretic. It makes use of previous categorical constructions from other parts of the book.



        For a more geometric proof you can refer to Deligne's original proof, which is actually exposed in detail in Johnstone's other book "Topos theory", section 7.4. It is actually essentially the same argument, though you can distinguish the geometric flavour here.



        And there is of course the proof given in Maclane-Moerdijk "Sheaves in geometry and logic" which is based on Barr's theorem and is also quite geometric.






        share|cite|improve this answer











        $endgroup$



        The proof you cite is certainly based in the completeness theorem for coherent logic, but that book contains a fully categorical proof of such a theorem, based on ideas of Joyal, so it can qualify as category-theoretic. It makes use of previous categorical constructions from other parts of the book.



        For a more geometric proof you can refer to Deligne's original proof, which is actually exposed in detail in Johnstone's other book "Topos theory", section 7.4. It is actually essentially the same argument, though you can distinguish the geometric flavour here.



        And there is of course the proof given in Maclane-Moerdijk "Sheaves in geometry and logic" which is based on Barr's theorem and is also quite geometric.







        share|cite|improve this answer














        share|cite|improve this answer



        share|cite|improve this answer








        edited 8 hours ago

























        answered 8 hours ago









        godeliangodelian

        2,7741 gold badge23 silver badges25 bronze badges




        2,7741 gold badge23 silver badges25 bronze badges






























            draft saved

            draft discarded
















































            Thanks for contributing an answer to MathOverflow!


            • Please be sure to answer the question. Provide details and share your research!

            But avoid


            • Asking for help, clarification, or responding to other answers.

            • Making statements based on opinion; back them up with references or personal experience.

            Use MathJax to format equations. MathJax reference.


            To learn more, see our tips on writing great answers.




            draft saved


            draft discarded














            StackExchange.ready(
            function ()
            StackExchange.openid.initPostLogin('.new-post-login', 'https%3a%2f%2fmathoverflow.net%2fquestions%2f338862%2fa-list-of-proofs-of-coherent-topoi-have-enough-points%23new-answer', 'question_page');

            );

            Post as a guest















            Required, but never shown





















































            Required, but never shown














            Required, but never shown












            Required, but never shown







            Required, but never shown

































            Required, but never shown














            Required, but never shown












            Required, but never shown







            Required, but never shown







            Popular posts from this blog

            Invision Community Contents History See also References External links Navigation menuProprietaryinvisioncommunity.comIPS Community ForumsIPS Community Forumsthis blog entry"License Changes, IP.Board 3.4, and the Future""Interview -- Matt Mecham of Ibforums""CEO Invision Power Board, Matt Mecham Is a Liar, Thief!"IPB License Explanation 1.3, 1.3.1, 2.0, and 2.1ArchivedSecurity Fixes, Updates And Enhancements For IPB 1.3.1Archived"New Demo Accounts - Invision Power Services"the original"New Default Skin"the original"Invision Power Board 3.0.0 and Applications Released"the original"Archived copy"the original"Perpetual licenses being done away with""Release Notes - Invision Power Services""Introducing: IPS Community Suite 4!"Invision Community Release Notes

            Canceling a color specificationRandomly assigning color to Graphics3D objects?Default color for Filling in Mathematica 9Coloring specific elements of sets with a prime modified order in an array plotHow to pick a color differing significantly from the colors already in a given color list?Detection of the text colorColor numbers based on their valueCan color schemes for use with ColorData include opacity specification?My dynamic color schemes

            Tom Holland Mục lục Đầu đời và giáo dục | Sự nghiệp | Cuộc sống cá nhân | Phim tham gia | Giải thưởng và đề cử | Chú thích | Liên kết ngoài | Trình đơn chuyển hướngProfile“Person Details for Thomas Stanley Holland, "England and Wales Birth Registration Index, 1837-2008" — FamilySearch.org”"Meet Tom Holland... the 16-year-old star of The Impossible""Schoolboy actor Tom Holland finds himself in Oscar contention for role in tsunami drama"“Naomi Watts on the Prince William and Harry's reaction to her film about the late Princess Diana”lưu trữ"Holland and Pflueger Are West End's Two New 'Billy Elliots'""I'm so envious of my son, the movie star! British writer Dominic Holland's spent 20 years trying to crack Hollywood - but he's been beaten to it by a very unlikely rival"“Richard and Margaret Povey of Jersey, Channel Islands, UK: Information about Thomas Stanley Holland”"Tom Holland to play Billy Elliot""New Billy Elliot leaving the garage"Billy Elliot the Musical - Tom Holland - Billy"A Tale of four Billys: Tom Holland""The Feel Good Factor""Thames Christian College schoolboys join Myleene Klass for The Feelgood Factor""Government launches £600,000 arts bursaries pilot""BILLY's Chapman, Holland, Gardner & Jackson-Keen Visit Prime Minister""Elton John 'blown away' by Billy Elliot fifth birthday" (video with John's interview and fragments of Holland's performance)"First News interviews Arrietty's Tom Holland"“33rd Critics' Circle Film Awards winners”“National Board of Review Current Awards”Bản gốc"Ron Howard Whaling Tale 'In The Heart Of The Sea' Casts Tom Holland"“'Spider-Man' Finds Tom Holland to Star as New Web-Slinger”lưu trữ“Captain America: Civil War (2016)”“Film Review: ‘Captain America: Civil War’”lưu trữ“‘Captain America: Civil War’ review: Choose your own avenger”lưu trữ“The Lost City of Z reviews”“Sony Pictures and Marvel Studios Find Their 'Spider-Man' Star and Director”“‘Mary Magdalene’, ‘Current War’ & ‘Wind River’ Get 2017 Release Dates From Weinstein”“Lionsgate Unleashing Daisy Ridley & Tom Holland Starrer ‘Chaos Walking’ In Cannes”“PTA's 'Master' Leads Chicago Film Critics Nominations, UPDATED: Houston and Indiana Critics Nominations”“Nominaciones Goya 2013 Telecinco Cinema – ENG”“Jameson Empire Film Awards: Martin Freeman wins best actor for performance in The Hobbit”“34th Annual Young Artist Awards”Bản gốc“Teen Choice Awards 2016—Captain America: Civil War Leads Second Wave of Nominations”“BAFTA Film Award Nominations: ‘La La Land’ Leads Race”“Saturn Awards Nominations 2017: 'Rogue One,' 'Walking Dead' Lead”Tom HollandTom HollandTom HollandTom Hollandmedia.gettyimages.comWorldCat Identities300279794no20130442900000 0004 0355 42791085670554170004732cb16706349t(data)XX5557367